Calculate inverse of matrix with -1 on diagonal and 1 on the rest












1












$begingroup$


Calculate the inverse of the matrix



begin{bmatrix}
-1& 1& ...& ...&1 \
1& -1& 1& ... &1 \
...& ...& ...& ...&1 \
1&1 &1 & ...&1 \
1& 1 &1 &1 &-1
end{bmatrix}



$-1$ on the diagonal and $1$ on the rest.



The key I think is to perform a sequence of elementary transformations on



[ A | $I_{n}$ ] until we get [ $I_{n}$ | $A^{-1}$ ] but that seems to be complicated.










share|cite|improve this question









$endgroup$












  • $begingroup$
    Your matrix can be expressed as $A+uv^top$ (see this answer. Then use the Sherman-Morrison formula.
    $endgroup$
    – StubbornAtom
    Nov 30 '18 at 18:50
















1












$begingroup$


Calculate the inverse of the matrix



begin{bmatrix}
-1& 1& ...& ...&1 \
1& -1& 1& ... &1 \
...& ...& ...& ...&1 \
1&1 &1 & ...&1 \
1& 1 &1 &1 &-1
end{bmatrix}



$-1$ on the diagonal and $1$ on the rest.



The key I think is to perform a sequence of elementary transformations on



[ A | $I_{n}$ ] until we get [ $I_{n}$ | $A^{-1}$ ] but that seems to be complicated.










share|cite|improve this question









$endgroup$












  • $begingroup$
    Your matrix can be expressed as $A+uv^top$ (see this answer. Then use the Sherman-Morrison formula.
    $endgroup$
    – StubbornAtom
    Nov 30 '18 at 18:50














1












1








1


1



$begingroup$


Calculate the inverse of the matrix



begin{bmatrix}
-1& 1& ...& ...&1 \
1& -1& 1& ... &1 \
...& ...& ...& ...&1 \
1&1 &1 & ...&1 \
1& 1 &1 &1 &-1
end{bmatrix}



$-1$ on the diagonal and $1$ on the rest.



The key I think is to perform a sequence of elementary transformations on



[ A | $I_{n}$ ] until we get [ $I_{n}$ | $A^{-1}$ ] but that seems to be complicated.










share|cite|improve this question









$endgroup$




Calculate the inverse of the matrix



begin{bmatrix}
-1& 1& ...& ...&1 \
1& -1& 1& ... &1 \
...& ...& ...& ...&1 \
1&1 &1 & ...&1 \
1& 1 &1 &1 &-1
end{bmatrix}



$-1$ on the diagonal and $1$ on the rest.



The key I think is to perform a sequence of elementary transformations on



[ A | $I_{n}$ ] until we get [ $I_{n}$ | $A^{-1}$ ] but that seems to be complicated.







matrices inverse






share|cite|improve this question













share|cite|improve this question











share|cite|improve this question




share|cite|improve this question










asked Nov 30 '18 at 18:34









SADBOYSSADBOYS

4288




4288












  • $begingroup$
    Your matrix can be expressed as $A+uv^top$ (see this answer. Then use the Sherman-Morrison formula.
    $endgroup$
    – StubbornAtom
    Nov 30 '18 at 18:50


















  • $begingroup$
    Your matrix can be expressed as $A+uv^top$ (see this answer. Then use the Sherman-Morrison formula.
    $endgroup$
    – StubbornAtom
    Nov 30 '18 at 18:50
















$begingroup$
Your matrix can be expressed as $A+uv^top$ (see this answer. Then use the Sherman-Morrison formula.
$endgroup$
– StubbornAtom
Nov 30 '18 at 18:50




$begingroup$
Your matrix can be expressed as $A+uv^top$ (see this answer. Then use the Sherman-Morrison formula.
$endgroup$
– StubbornAtom
Nov 30 '18 at 18:50










3 Answers
3






active

oldest

votes


















3












$begingroup$

Let $e$ be the all one vector. We have



$$A=-2I+ee^T$$
By Sheman-Morrison formula:
begin{align}A^{-1}&=(-2I+ee^T)^{-1}\&=-frac12I-frac{-left(frac12Iright)ee^Tleft(-frac12Iright)}{1+e^Tleft( -frac12Iright)e} \
&=-frac12I-frac{frac14ee^T}{1-frac{n}2} \
&=-frac12I-frac{ee^T}{4-2n}
end{align}



Hence, the off diagonal entries are $-frac1{4-2n}$ and the diagonal entries are $-frac12-frac1{4-2n}$.



Remark: If $n=2$, the matrix is not invertible.






share|cite|improve this answer









$endgroup$













  • $begingroup$
    Because the determinant is $-2^{n-1}(n-2)$ I think, thanks for the help^^
    $endgroup$
    – SADBOYS
    Nov 30 '18 at 22:56



















2












$begingroup$

The inverse also has a constant $a$ on the main diagonal and $b$ everywhere else. The constants do depend on the matrix size $n;$ as noted, for $n=2$ there is no inverse. Just try the thing for $n=3$ and $n=4$ and $n=5.$ By that time you should have it, for $n geq 3$



for $n=3:$
$$
left(
begin{array}{rrr}
-1 & 1&1 \
1 & -1&1 \
1 & 1&-1 \
end{array}
right)
left(
begin{array}{ccc}
a & b&b \
b & a&b \
b & b&a \
end{array}
right)=
left(
begin{array}{rrr}
1 & 0&0 \
0 & 1&0 \
0 & 0&1 \
end{array}
right)
$$



for $n=4,$ different $a,b:$
$$
left(
begin{array}{rrrr}
-1 & 1&1&1 \
1 & -1&1&1 \
1 & 1&-1&1 \
1 & 1&1&-1 \
end{array}
right)
left(
begin{array}{cccc}
a & b&b&b \
b & a&b&b \
b & b&a&b \
b & b&b&a \
end{array}
right)=
left(
begin{array}{rrrr}
1 & 0&0&0 \
0 & 1&0&0 \
0 & 0&1&0 \
0 & 0&0&1 \
end{array}
right)
$$



for $n=5,$ still different $a,b:$
$$
left(
begin{array}{rrrrr}
-1 & 1&1&1&1 \
1 & -1&1&1&1 \
1 & 1&-1&1&1 \
1 & 1&1&-1&1 \
1&1&1&1&-1 \
end{array}
right)
left(
begin{array}{ccccc}
a & b&b&b &b\
b & a&b&b&b \
b & b&a&b &b \
b & b&b&a &b \
b&b&b&b&a \
end{array}
right)=
left(
begin{array}{rrrrr}
1 & 0&0&0 &0 \
0 & 1&0&0&0 \
0 & 0&1&0 &0 \
0 & 0&0&1 &0 \
0&0&0&0&1
end{array}
right)
$$






share|cite|improve this answer











$endgroup$





















    1












    $begingroup$

    It's not too bad...
    $$begin{array}{c}-1\-1\vdots\*
    end{array}left[begin{array}{cccc|cccc}
    -1&1&cdots&1 &1\
    1&-1&cdots&1 &&1\
    vdots&vdots&ddots&vdots &&&ddots\
    1&1&cdots&-1 &&&&1
    end{array}right] implies$$



    $$begin{array}{c}*\*\vdots\small 1/2end{array}
    left[begin{array}{cccc|cccc}
    -2&&&2 &1&&&-1\
    &-2&&2 &&1&&-1\
    &&ddots& &&&ddots\
    1&1&&-1 &&&&1
    end{array}right] implies$$



    $$begin{array}{c}*small -1/2\*small -1/2\vdots\* small ^1!/_{!n-2}end{array}
    left[begin{array}{cccc|cccc}
    -2&&&2 &1&&&-1\
    &-2&&2 &&1&&-1\
    &&ddots& &&&ddots\
    &&&n-2 &small 1/2&small 1/2&&small-^{(n-3)!}/_{!2}
    end{array}right] implies$$



    $$begin{array}{c}1\1\vdots\*end{array}
    left[begin{array}{cccc|cccc}
    1&&&-1 &small -1/2&&&small 1/2\
    &1&&-1 &&small -1/2&&small 1/2\
    &&ddots& &&&ddots\
    &&&1 &small ^1!/_{!2(n-2)}&small ^1!/_{!2(n-2)}&&small -^1!/_{!2}+ ^1!/_{!2(n-2)}
    end{array}right] implies$$



    $$begin{array}{c} \ \ \ end{array}
    left[begin{array}{cccc|cccc}
    1&&& &small -^1!/_{!2}+ ^1!/_{!2(n-2)}&small ^1!/_{!2(n-2)}&cdots&small ^1!/_{!2(n-2)}\
    &1&& &small ^1!/_{!2(n-2)}&small -^1!/_{!2}+ ^1!/_{!2(n-2)}&cdots&small ^1!/_{!2(n-2)}\
    &&ddots& &vdots&vdots&ddots&vdots\
    &&&1 &small ^1!/_{!2(n-2)}&small ^1!/_{!2(n-2)}&cdots&small -^1!/_{!2}+ ^1!/_{!2(n-2)}
    end{array}right]$$






    share|cite|improve this answer









    $endgroup$













      Your Answer





      StackExchange.ifUsing("editor", function () {
      return StackExchange.using("mathjaxEditing", function () {
      StackExchange.MarkdownEditor.creationCallbacks.add(function (editor, postfix) {
      StackExchange.mathjaxEditing.prepareWmdForMathJax(editor, postfix, [["$", "$"], ["\\(","\\)"]]);
      });
      });
      }, "mathjax-editing");

      StackExchange.ready(function() {
      var channelOptions = {
      tags: "".split(" "),
      id: "69"
      };
      initTagRenderer("".split(" "), "".split(" "), channelOptions);

      StackExchange.using("externalEditor", function() {
      // Have to fire editor after snippets, if snippets enabled
      if (StackExchange.settings.snippets.snippetsEnabled) {
      StackExchange.using("snippets", function() {
      createEditor();
      });
      }
      else {
      createEditor();
      }
      });

      function createEditor() {
      StackExchange.prepareEditor({
      heartbeatType: 'answer',
      autoActivateHeartbeat: false,
      convertImagesToLinks: true,
      noModals: true,
      showLowRepImageUploadWarning: true,
      reputationToPostImages: 10,
      bindNavPrevention: true,
      postfix: "",
      imageUploader: {
      brandingHtml: "Powered by u003ca class="icon-imgur-white" href="https://imgur.com/"u003eu003c/au003e",
      contentPolicyHtml: "User contributions licensed under u003ca href="https://creativecommons.org/licenses/by-sa/3.0/"u003ecc by-sa 3.0 with attribution requiredu003c/au003e u003ca href="https://stackoverflow.com/legal/content-policy"u003e(content policy)u003c/au003e",
      allowUrls: true
      },
      noCode: true, onDemand: true,
      discardSelector: ".discard-answer"
      ,immediatelyShowMarkdownHelp:true
      });


      }
      });














      draft saved

      draft discarded


















      StackExchange.ready(
      function () {
      StackExchange.openid.initPostLogin('.new-post-login', 'https%3a%2f%2fmath.stackexchange.com%2fquestions%2f3020465%2fcalculate-inverse-of-matrix-with-1-on-diagonal-and-1-on-the-rest%23new-answer', 'question_page');
      }
      );

      Post as a guest















      Required, but never shown

























      3 Answers
      3






      active

      oldest

      votes








      3 Answers
      3






      active

      oldest

      votes









      active

      oldest

      votes






      active

      oldest

      votes









      3












      $begingroup$

      Let $e$ be the all one vector. We have



      $$A=-2I+ee^T$$
      By Sheman-Morrison formula:
      begin{align}A^{-1}&=(-2I+ee^T)^{-1}\&=-frac12I-frac{-left(frac12Iright)ee^Tleft(-frac12Iright)}{1+e^Tleft( -frac12Iright)e} \
      &=-frac12I-frac{frac14ee^T}{1-frac{n}2} \
      &=-frac12I-frac{ee^T}{4-2n}
      end{align}



      Hence, the off diagonal entries are $-frac1{4-2n}$ and the diagonal entries are $-frac12-frac1{4-2n}$.



      Remark: If $n=2$, the matrix is not invertible.






      share|cite|improve this answer









      $endgroup$













      • $begingroup$
        Because the determinant is $-2^{n-1}(n-2)$ I think, thanks for the help^^
        $endgroup$
        – SADBOYS
        Nov 30 '18 at 22:56
















      3












      $begingroup$

      Let $e$ be the all one vector. We have



      $$A=-2I+ee^T$$
      By Sheman-Morrison formula:
      begin{align}A^{-1}&=(-2I+ee^T)^{-1}\&=-frac12I-frac{-left(frac12Iright)ee^Tleft(-frac12Iright)}{1+e^Tleft( -frac12Iright)e} \
      &=-frac12I-frac{frac14ee^T}{1-frac{n}2} \
      &=-frac12I-frac{ee^T}{4-2n}
      end{align}



      Hence, the off diagonal entries are $-frac1{4-2n}$ and the diagonal entries are $-frac12-frac1{4-2n}$.



      Remark: If $n=2$, the matrix is not invertible.






      share|cite|improve this answer









      $endgroup$













      • $begingroup$
        Because the determinant is $-2^{n-1}(n-2)$ I think, thanks for the help^^
        $endgroup$
        – SADBOYS
        Nov 30 '18 at 22:56














      3












      3








      3





      $begingroup$

      Let $e$ be the all one vector. We have



      $$A=-2I+ee^T$$
      By Sheman-Morrison formula:
      begin{align}A^{-1}&=(-2I+ee^T)^{-1}\&=-frac12I-frac{-left(frac12Iright)ee^Tleft(-frac12Iright)}{1+e^Tleft( -frac12Iright)e} \
      &=-frac12I-frac{frac14ee^T}{1-frac{n}2} \
      &=-frac12I-frac{ee^T}{4-2n}
      end{align}



      Hence, the off diagonal entries are $-frac1{4-2n}$ and the diagonal entries are $-frac12-frac1{4-2n}$.



      Remark: If $n=2$, the matrix is not invertible.






      share|cite|improve this answer









      $endgroup$



      Let $e$ be the all one vector. We have



      $$A=-2I+ee^T$$
      By Sheman-Morrison formula:
      begin{align}A^{-1}&=(-2I+ee^T)^{-1}\&=-frac12I-frac{-left(frac12Iright)ee^Tleft(-frac12Iright)}{1+e^Tleft( -frac12Iright)e} \
      &=-frac12I-frac{frac14ee^T}{1-frac{n}2} \
      &=-frac12I-frac{ee^T}{4-2n}
      end{align}



      Hence, the off diagonal entries are $-frac1{4-2n}$ and the diagonal entries are $-frac12-frac1{4-2n}$.



      Remark: If $n=2$, the matrix is not invertible.







      share|cite|improve this answer












      share|cite|improve this answer



      share|cite|improve this answer










      answered Nov 30 '18 at 18:46









      Siong Thye GohSiong Thye Goh

      100k1465117




      100k1465117












      • $begingroup$
        Because the determinant is $-2^{n-1}(n-2)$ I think, thanks for the help^^
        $endgroup$
        – SADBOYS
        Nov 30 '18 at 22:56


















      • $begingroup$
        Because the determinant is $-2^{n-1}(n-2)$ I think, thanks for the help^^
        $endgroup$
        – SADBOYS
        Nov 30 '18 at 22:56
















      $begingroup$
      Because the determinant is $-2^{n-1}(n-2)$ I think, thanks for the help^^
      $endgroup$
      – SADBOYS
      Nov 30 '18 at 22:56




      $begingroup$
      Because the determinant is $-2^{n-1}(n-2)$ I think, thanks for the help^^
      $endgroup$
      – SADBOYS
      Nov 30 '18 at 22:56











      2












      $begingroup$

      The inverse also has a constant $a$ on the main diagonal and $b$ everywhere else. The constants do depend on the matrix size $n;$ as noted, for $n=2$ there is no inverse. Just try the thing for $n=3$ and $n=4$ and $n=5.$ By that time you should have it, for $n geq 3$



      for $n=3:$
      $$
      left(
      begin{array}{rrr}
      -1 & 1&1 \
      1 & -1&1 \
      1 & 1&-1 \
      end{array}
      right)
      left(
      begin{array}{ccc}
      a & b&b \
      b & a&b \
      b & b&a \
      end{array}
      right)=
      left(
      begin{array}{rrr}
      1 & 0&0 \
      0 & 1&0 \
      0 & 0&1 \
      end{array}
      right)
      $$



      for $n=4,$ different $a,b:$
      $$
      left(
      begin{array}{rrrr}
      -1 & 1&1&1 \
      1 & -1&1&1 \
      1 & 1&-1&1 \
      1 & 1&1&-1 \
      end{array}
      right)
      left(
      begin{array}{cccc}
      a & b&b&b \
      b & a&b&b \
      b & b&a&b \
      b & b&b&a \
      end{array}
      right)=
      left(
      begin{array}{rrrr}
      1 & 0&0&0 \
      0 & 1&0&0 \
      0 & 0&1&0 \
      0 & 0&0&1 \
      end{array}
      right)
      $$



      for $n=5,$ still different $a,b:$
      $$
      left(
      begin{array}{rrrrr}
      -1 & 1&1&1&1 \
      1 & -1&1&1&1 \
      1 & 1&-1&1&1 \
      1 & 1&1&-1&1 \
      1&1&1&1&-1 \
      end{array}
      right)
      left(
      begin{array}{ccccc}
      a & b&b&b &b\
      b & a&b&b&b \
      b & b&a&b &b \
      b & b&b&a &b \
      b&b&b&b&a \
      end{array}
      right)=
      left(
      begin{array}{rrrrr}
      1 & 0&0&0 &0 \
      0 & 1&0&0&0 \
      0 & 0&1&0 &0 \
      0 & 0&0&1 &0 \
      0&0&0&0&1
      end{array}
      right)
      $$






      share|cite|improve this answer











      $endgroup$


















        2












        $begingroup$

        The inverse also has a constant $a$ on the main diagonal and $b$ everywhere else. The constants do depend on the matrix size $n;$ as noted, for $n=2$ there is no inverse. Just try the thing for $n=3$ and $n=4$ and $n=5.$ By that time you should have it, for $n geq 3$



        for $n=3:$
        $$
        left(
        begin{array}{rrr}
        -1 & 1&1 \
        1 & -1&1 \
        1 & 1&-1 \
        end{array}
        right)
        left(
        begin{array}{ccc}
        a & b&b \
        b & a&b \
        b & b&a \
        end{array}
        right)=
        left(
        begin{array}{rrr}
        1 & 0&0 \
        0 & 1&0 \
        0 & 0&1 \
        end{array}
        right)
        $$



        for $n=4,$ different $a,b:$
        $$
        left(
        begin{array}{rrrr}
        -1 & 1&1&1 \
        1 & -1&1&1 \
        1 & 1&-1&1 \
        1 & 1&1&-1 \
        end{array}
        right)
        left(
        begin{array}{cccc}
        a & b&b&b \
        b & a&b&b \
        b & b&a&b \
        b & b&b&a \
        end{array}
        right)=
        left(
        begin{array}{rrrr}
        1 & 0&0&0 \
        0 & 1&0&0 \
        0 & 0&1&0 \
        0 & 0&0&1 \
        end{array}
        right)
        $$



        for $n=5,$ still different $a,b:$
        $$
        left(
        begin{array}{rrrrr}
        -1 & 1&1&1&1 \
        1 & -1&1&1&1 \
        1 & 1&-1&1&1 \
        1 & 1&1&-1&1 \
        1&1&1&1&-1 \
        end{array}
        right)
        left(
        begin{array}{ccccc}
        a & b&b&b &b\
        b & a&b&b&b \
        b & b&a&b &b \
        b & b&b&a &b \
        b&b&b&b&a \
        end{array}
        right)=
        left(
        begin{array}{rrrrr}
        1 & 0&0&0 &0 \
        0 & 1&0&0&0 \
        0 & 0&1&0 &0 \
        0 & 0&0&1 &0 \
        0&0&0&0&1
        end{array}
        right)
        $$






        share|cite|improve this answer











        $endgroup$
















          2












          2








          2





          $begingroup$

          The inverse also has a constant $a$ on the main diagonal and $b$ everywhere else. The constants do depend on the matrix size $n;$ as noted, for $n=2$ there is no inverse. Just try the thing for $n=3$ and $n=4$ and $n=5.$ By that time you should have it, for $n geq 3$



          for $n=3:$
          $$
          left(
          begin{array}{rrr}
          -1 & 1&1 \
          1 & -1&1 \
          1 & 1&-1 \
          end{array}
          right)
          left(
          begin{array}{ccc}
          a & b&b \
          b & a&b \
          b & b&a \
          end{array}
          right)=
          left(
          begin{array}{rrr}
          1 & 0&0 \
          0 & 1&0 \
          0 & 0&1 \
          end{array}
          right)
          $$



          for $n=4,$ different $a,b:$
          $$
          left(
          begin{array}{rrrr}
          -1 & 1&1&1 \
          1 & -1&1&1 \
          1 & 1&-1&1 \
          1 & 1&1&-1 \
          end{array}
          right)
          left(
          begin{array}{cccc}
          a & b&b&b \
          b & a&b&b \
          b & b&a&b \
          b & b&b&a \
          end{array}
          right)=
          left(
          begin{array}{rrrr}
          1 & 0&0&0 \
          0 & 1&0&0 \
          0 & 0&1&0 \
          0 & 0&0&1 \
          end{array}
          right)
          $$



          for $n=5,$ still different $a,b:$
          $$
          left(
          begin{array}{rrrrr}
          -1 & 1&1&1&1 \
          1 & -1&1&1&1 \
          1 & 1&-1&1&1 \
          1 & 1&1&-1&1 \
          1&1&1&1&-1 \
          end{array}
          right)
          left(
          begin{array}{ccccc}
          a & b&b&b &b\
          b & a&b&b&b \
          b & b&a&b &b \
          b & b&b&a &b \
          b&b&b&b&a \
          end{array}
          right)=
          left(
          begin{array}{rrrrr}
          1 & 0&0&0 &0 \
          0 & 1&0&0&0 \
          0 & 0&1&0 &0 \
          0 & 0&0&1 &0 \
          0&0&0&0&1
          end{array}
          right)
          $$






          share|cite|improve this answer











          $endgroup$



          The inverse also has a constant $a$ on the main diagonal and $b$ everywhere else. The constants do depend on the matrix size $n;$ as noted, for $n=2$ there is no inverse. Just try the thing for $n=3$ and $n=4$ and $n=5.$ By that time you should have it, for $n geq 3$



          for $n=3:$
          $$
          left(
          begin{array}{rrr}
          -1 & 1&1 \
          1 & -1&1 \
          1 & 1&-1 \
          end{array}
          right)
          left(
          begin{array}{ccc}
          a & b&b \
          b & a&b \
          b & b&a \
          end{array}
          right)=
          left(
          begin{array}{rrr}
          1 & 0&0 \
          0 & 1&0 \
          0 & 0&1 \
          end{array}
          right)
          $$



          for $n=4,$ different $a,b:$
          $$
          left(
          begin{array}{rrrr}
          -1 & 1&1&1 \
          1 & -1&1&1 \
          1 & 1&-1&1 \
          1 & 1&1&-1 \
          end{array}
          right)
          left(
          begin{array}{cccc}
          a & b&b&b \
          b & a&b&b \
          b & b&a&b \
          b & b&b&a \
          end{array}
          right)=
          left(
          begin{array}{rrrr}
          1 & 0&0&0 \
          0 & 1&0&0 \
          0 & 0&1&0 \
          0 & 0&0&1 \
          end{array}
          right)
          $$



          for $n=5,$ still different $a,b:$
          $$
          left(
          begin{array}{rrrrr}
          -1 & 1&1&1&1 \
          1 & -1&1&1&1 \
          1 & 1&-1&1&1 \
          1 & 1&1&-1&1 \
          1&1&1&1&-1 \
          end{array}
          right)
          left(
          begin{array}{ccccc}
          a & b&b&b &b\
          b & a&b&b&b \
          b & b&a&b &b \
          b & b&b&a &b \
          b&b&b&b&a \
          end{array}
          right)=
          left(
          begin{array}{rrrrr}
          1 & 0&0&0 &0 \
          0 & 1&0&0&0 \
          0 & 0&1&0 &0 \
          0 & 0&0&1 &0 \
          0&0&0&0&1
          end{array}
          right)
          $$







          share|cite|improve this answer














          share|cite|improve this answer



          share|cite|improve this answer








          edited Nov 30 '18 at 19:21

























          answered Nov 30 '18 at 19:04









          Will JagyWill Jagy

          102k5100199




          102k5100199























              1












              $begingroup$

              It's not too bad...
              $$begin{array}{c}-1\-1\vdots\*
              end{array}left[begin{array}{cccc|cccc}
              -1&1&cdots&1 &1\
              1&-1&cdots&1 &&1\
              vdots&vdots&ddots&vdots &&&ddots\
              1&1&cdots&-1 &&&&1
              end{array}right] implies$$



              $$begin{array}{c}*\*\vdots\small 1/2end{array}
              left[begin{array}{cccc|cccc}
              -2&&&2 &1&&&-1\
              &-2&&2 &&1&&-1\
              &&ddots& &&&ddots\
              1&1&&-1 &&&&1
              end{array}right] implies$$



              $$begin{array}{c}*small -1/2\*small -1/2\vdots\* small ^1!/_{!n-2}end{array}
              left[begin{array}{cccc|cccc}
              -2&&&2 &1&&&-1\
              &-2&&2 &&1&&-1\
              &&ddots& &&&ddots\
              &&&n-2 &small 1/2&small 1/2&&small-^{(n-3)!}/_{!2}
              end{array}right] implies$$



              $$begin{array}{c}1\1\vdots\*end{array}
              left[begin{array}{cccc|cccc}
              1&&&-1 &small -1/2&&&small 1/2\
              &1&&-1 &&small -1/2&&small 1/2\
              &&ddots& &&&ddots\
              &&&1 &small ^1!/_{!2(n-2)}&small ^1!/_{!2(n-2)}&&small -^1!/_{!2}+ ^1!/_{!2(n-2)}
              end{array}right] implies$$



              $$begin{array}{c} \ \ \ end{array}
              left[begin{array}{cccc|cccc}
              1&&& &small -^1!/_{!2}+ ^1!/_{!2(n-2)}&small ^1!/_{!2(n-2)}&cdots&small ^1!/_{!2(n-2)}\
              &1&& &small ^1!/_{!2(n-2)}&small -^1!/_{!2}+ ^1!/_{!2(n-2)}&cdots&small ^1!/_{!2(n-2)}\
              &&ddots& &vdots&vdots&ddots&vdots\
              &&&1 &small ^1!/_{!2(n-2)}&small ^1!/_{!2(n-2)}&cdots&small -^1!/_{!2}+ ^1!/_{!2(n-2)}
              end{array}right]$$






              share|cite|improve this answer









              $endgroup$


















                1












                $begingroup$

                It's not too bad...
                $$begin{array}{c}-1\-1\vdots\*
                end{array}left[begin{array}{cccc|cccc}
                -1&1&cdots&1 &1\
                1&-1&cdots&1 &&1\
                vdots&vdots&ddots&vdots &&&ddots\
                1&1&cdots&-1 &&&&1
                end{array}right] implies$$



                $$begin{array}{c}*\*\vdots\small 1/2end{array}
                left[begin{array}{cccc|cccc}
                -2&&&2 &1&&&-1\
                &-2&&2 &&1&&-1\
                &&ddots& &&&ddots\
                1&1&&-1 &&&&1
                end{array}right] implies$$



                $$begin{array}{c}*small -1/2\*small -1/2\vdots\* small ^1!/_{!n-2}end{array}
                left[begin{array}{cccc|cccc}
                -2&&&2 &1&&&-1\
                &-2&&2 &&1&&-1\
                &&ddots& &&&ddots\
                &&&n-2 &small 1/2&small 1/2&&small-^{(n-3)!}/_{!2}
                end{array}right] implies$$



                $$begin{array}{c}1\1\vdots\*end{array}
                left[begin{array}{cccc|cccc}
                1&&&-1 &small -1/2&&&small 1/2\
                &1&&-1 &&small -1/2&&small 1/2\
                &&ddots& &&&ddots\
                &&&1 &small ^1!/_{!2(n-2)}&small ^1!/_{!2(n-2)}&&small -^1!/_{!2}+ ^1!/_{!2(n-2)}
                end{array}right] implies$$



                $$begin{array}{c} \ \ \ end{array}
                left[begin{array}{cccc|cccc}
                1&&& &small -^1!/_{!2}+ ^1!/_{!2(n-2)}&small ^1!/_{!2(n-2)}&cdots&small ^1!/_{!2(n-2)}\
                &1&& &small ^1!/_{!2(n-2)}&small -^1!/_{!2}+ ^1!/_{!2(n-2)}&cdots&small ^1!/_{!2(n-2)}\
                &&ddots& &vdots&vdots&ddots&vdots\
                &&&1 &small ^1!/_{!2(n-2)}&small ^1!/_{!2(n-2)}&cdots&small -^1!/_{!2}+ ^1!/_{!2(n-2)}
                end{array}right]$$






                share|cite|improve this answer









                $endgroup$
















                  1












                  1








                  1





                  $begingroup$

                  It's not too bad...
                  $$begin{array}{c}-1\-1\vdots\*
                  end{array}left[begin{array}{cccc|cccc}
                  -1&1&cdots&1 &1\
                  1&-1&cdots&1 &&1\
                  vdots&vdots&ddots&vdots &&&ddots\
                  1&1&cdots&-1 &&&&1
                  end{array}right] implies$$



                  $$begin{array}{c}*\*\vdots\small 1/2end{array}
                  left[begin{array}{cccc|cccc}
                  -2&&&2 &1&&&-1\
                  &-2&&2 &&1&&-1\
                  &&ddots& &&&ddots\
                  1&1&&-1 &&&&1
                  end{array}right] implies$$



                  $$begin{array}{c}*small -1/2\*small -1/2\vdots\* small ^1!/_{!n-2}end{array}
                  left[begin{array}{cccc|cccc}
                  -2&&&2 &1&&&-1\
                  &-2&&2 &&1&&-1\
                  &&ddots& &&&ddots\
                  &&&n-2 &small 1/2&small 1/2&&small-^{(n-3)!}/_{!2}
                  end{array}right] implies$$



                  $$begin{array}{c}1\1\vdots\*end{array}
                  left[begin{array}{cccc|cccc}
                  1&&&-1 &small -1/2&&&small 1/2\
                  &1&&-1 &&small -1/2&&small 1/2\
                  &&ddots& &&&ddots\
                  &&&1 &small ^1!/_{!2(n-2)}&small ^1!/_{!2(n-2)}&&small -^1!/_{!2}+ ^1!/_{!2(n-2)}
                  end{array}right] implies$$



                  $$begin{array}{c} \ \ \ end{array}
                  left[begin{array}{cccc|cccc}
                  1&&& &small -^1!/_{!2}+ ^1!/_{!2(n-2)}&small ^1!/_{!2(n-2)}&cdots&small ^1!/_{!2(n-2)}\
                  &1&& &small ^1!/_{!2(n-2)}&small -^1!/_{!2}+ ^1!/_{!2(n-2)}&cdots&small ^1!/_{!2(n-2)}\
                  &&ddots& &vdots&vdots&ddots&vdots\
                  &&&1 &small ^1!/_{!2(n-2)}&small ^1!/_{!2(n-2)}&cdots&small -^1!/_{!2}+ ^1!/_{!2(n-2)}
                  end{array}right]$$






                  share|cite|improve this answer









                  $endgroup$



                  It's not too bad...
                  $$begin{array}{c}-1\-1\vdots\*
                  end{array}left[begin{array}{cccc|cccc}
                  -1&1&cdots&1 &1\
                  1&-1&cdots&1 &&1\
                  vdots&vdots&ddots&vdots &&&ddots\
                  1&1&cdots&-1 &&&&1
                  end{array}right] implies$$



                  $$begin{array}{c}*\*\vdots\small 1/2end{array}
                  left[begin{array}{cccc|cccc}
                  -2&&&2 &1&&&-1\
                  &-2&&2 &&1&&-1\
                  &&ddots& &&&ddots\
                  1&1&&-1 &&&&1
                  end{array}right] implies$$



                  $$begin{array}{c}*small -1/2\*small -1/2\vdots\* small ^1!/_{!n-2}end{array}
                  left[begin{array}{cccc|cccc}
                  -2&&&2 &1&&&-1\
                  &-2&&2 &&1&&-1\
                  &&ddots& &&&ddots\
                  &&&n-2 &small 1/2&small 1/2&&small-^{(n-3)!}/_{!2}
                  end{array}right] implies$$



                  $$begin{array}{c}1\1\vdots\*end{array}
                  left[begin{array}{cccc|cccc}
                  1&&&-1 &small -1/2&&&small 1/2\
                  &1&&-1 &&small -1/2&&small 1/2\
                  &&ddots& &&&ddots\
                  &&&1 &small ^1!/_{!2(n-2)}&small ^1!/_{!2(n-2)}&&small -^1!/_{!2}+ ^1!/_{!2(n-2)}
                  end{array}right] implies$$



                  $$begin{array}{c} \ \ \ end{array}
                  left[begin{array}{cccc|cccc}
                  1&&& &small -^1!/_{!2}+ ^1!/_{!2(n-2)}&small ^1!/_{!2(n-2)}&cdots&small ^1!/_{!2(n-2)}\
                  &1&& &small ^1!/_{!2(n-2)}&small -^1!/_{!2}+ ^1!/_{!2(n-2)}&cdots&small ^1!/_{!2(n-2)}\
                  &&ddots& &vdots&vdots&ddots&vdots\
                  &&&1 &small ^1!/_{!2(n-2)}&small ^1!/_{!2(n-2)}&cdots&small -^1!/_{!2}+ ^1!/_{!2(n-2)}
                  end{array}right]$$







                  share|cite|improve this answer












                  share|cite|improve this answer



                  share|cite|improve this answer










                  answered Nov 30 '18 at 20:30









                  I like SerenaI like Serena

                  3,7471718




                  3,7471718






























                      draft saved

                      draft discarded




















































                      Thanks for contributing an answer to Mathematics Stack Exchange!


                      • Please be sure to answer the question. Provide details and share your research!

                      But avoid



                      • Asking for help, clarification, or responding to other answers.

                      • Making statements based on opinion; back them up with references or personal experience.


                      Use MathJax to format equations. MathJax reference.


                      To learn more, see our tips on writing great answers.




                      draft saved


                      draft discarded














                      StackExchange.ready(
                      function () {
                      StackExchange.openid.initPostLogin('.new-post-login', 'https%3a%2f%2fmath.stackexchange.com%2fquestions%2f3020465%2fcalculate-inverse-of-matrix-with-1-on-diagonal-and-1-on-the-rest%23new-answer', 'question_page');
                      }
                      );

                      Post as a guest















                      Required, but never shown





















































                      Required, but never shown














                      Required, but never shown












                      Required, but never shown







                      Required, but never shown

































                      Required, but never shown














                      Required, but never shown












                      Required, but never shown







                      Required, but never shown







                      Popular posts from this blog

                      Quarter-circle Tiles

                      build a pushdown automaton that recognizes the reverse language of a given pushdown automaton?

                      Mont Emei